RegistrierenRegistrieren   LoginLogin   FAQFAQ    SuchenSuchen   
Widerstand + Querschnittsfläche
 
Neue Frage »
Antworten »
    Foren-Übersicht -> Elektrik
Autor Nachricht
Grohte
Gast





Beitrag Grohte Verfasst am: 07. Okt 2011 23:04    Titel: Widerstand + Querschnittsfläche Antworten mit Zitat

Hallo

Ich möcht euch Fragen ob ihr es wisst was ich nehmen muss?

Ich habe einen Zylindrischen Wiederstend, an den Seitenflächen sind Kontakte befestigt die aber nur die Halbe Fläche der Seitenflächen des Wiederstandes haben.

wenn ich nach R = p*L/A gehe ist dann mit A die Fläche des Widerstandes oder die Fläche der Kontakte gemeint?

wenn die Annahme von (Der Strom nimt immer den Weg des geringsten Widerstandes ) nehem

würde ich die Fläche der Kontakte nehmen
Grund:
warum sollte der Strom denn ganzen Wiederstand ausfüllen wenn doch der geringste Weg von Kontakt zu Kontakt führt.
No_body



Anmeldungsdatum: 08.10.2011
Beiträge: 24

Beitrag No_body Verfasst am: 09. Okt 2011 11:42    Titel: Antworten mit Zitat

Mit A ist die Fläche des zylindrischen Widerstands gemeint. Für die Kontakte kann man auch einen Widerstand angeben, dort würde man dann aber A/2 verwenden. Die Kontakte an dem zylindrischen Widerstand haben somit grundsätzlich nichts mit dem Widerstand zwischen den beiden Kontakten zu tun.

In dem Zusammenhang kann man sich den Stromfluss analog zum Fluss von Wasser durch unterschiedlich breite Rohre vorstellen. Am Kontakt hat man eine Fläche von A/2 und am zylindrischen Widerstand von A, oder eben unterschiedlich Breite Rohre, wenn man das Analogon verwendet.
Grohte
Gast





Beitrag Grohte Verfasst am: 09. Okt 2011 14:01    Titel: Antworten mit Zitat

demnach spielt es keine Rolle wie groß die Fläche der Kontakte gegeüber der Fläche des Wiederstand ist?
No_body



Anmeldungsdatum: 08.10.2011
Beiträge: 24

Beitrag No_body Verfasst am: 09. Okt 2011 14:15    Titel: Antworten mit Zitat

Für den zylindrischen Widerstand zwischen den beiden Kotakten spielt die Querschnittsfläche der Kontakte nur einer vernachlässigbar kleine - also quasi keine - Rolle.

Wenn du ein Widerstands-System ganz exakt betrachten willst hast du zwei Kontakte und den eigentlichen Widerstand. Die Kontakte haben auch einen Widerstand, der ist aber meist vernachlässigbar klein. Für den Widerstand des Kontakts spielt seine Querschnittsfläche aber wieder eine Rolle.

Bei deiner Aufgabe würde ich aber davon ausgehn das die Kontaktwiderstände - wie üblich- vernachlässigt werden, deswegen ist dein A die Querschnittsfläche des zylindrischen Widerstands zwischen den Kontakten.
GvC



Anmeldungsdatum: 07.05.2009
Beiträge: 14861

Beitrag GvC Verfasst am: 09. Okt 2011 14:25    Titel: Antworten mit Zitat

Nein, nein, so geht das nicht. Wenn die Kontakte, deren Widerstand idealerweise erstmal zu Null angenommen wird, nur die halbe Querschnittsfläche berühren, dann fleißt der Strom auch nur über diese halbe Fläche in das Widerstandsmaterial hinein (und auch über die halbe Fläche hinaus). Dazwischen aber verteilt sich der Strom über die gesamte Querschnittsfläche. Im Übergangsbereich hast Du ein stark inhomogenes Feld, dessen Verlauf sich nur iterativ lösen lässt. Dazu gibt es entsprechende Feldberechnungsprogramme. Geschlossen kannst Du das jedenfalls nicht lösen.
Grohte
Gast





Beitrag Grohte Verfasst am: 09. Okt 2011 15:20    Titel: Antworten mit Zitat

Ich hätte gedacht der Wiederstand würde so sein
R = p*L/A/2

zylindrischen Widerstand
p =10^5 Ohm
L =0,1 Meter
A =7,854 m^2 (0,01m*0,01m*pi/4)
kontaktfläche = A/2
kontakte aus Aluminium

GvC
Dazwischen aber verteilt sich der Strom über die gesamte Querschnittsfläche

wieso?

müsste der Strom sich nicht den küzesten Weg wehlen die Grade genau zwischen den Kontakten, den engstligenden Feldlinen (wenn man den wiederstand weg nimmt so Kondensator ähnlich) entlang?
GvC



Anmeldungsdatum: 07.05.2009
Beiträge: 14861

Beitrag GvC Verfasst am: 09. Okt 2011 15:24    Titel: Antworten mit Zitat

Der Strom wählt nicht den kürzesten Weg, sondern den des geringsten Widerstandes. Und in den Widerstand geht nicht nur die Länge, sondern auch die Querschnittsfläche ein.
Grohte
Gast





Beitrag Grohte Verfasst am: 09. Okt 2011 18:30    Titel: Antworten mit Zitat

geringste Widerstand,

ist der ganze Widerstand, Fläche und Länge egal wo die Kontakte sind und egal wie groß die Kontate sind ob kleiner oder auch größer als die Fläche des Widerstandes zur Fläche des Kontate.

richtig?
No_body



Anmeldungsdatum: 08.10.2011
Beiträge: 24

Beitrag No_body Verfasst am: 09. Okt 2011 18:49    Titel: Antworten mit Zitat

Die Lage der Kontakte ist auch für eine vereinfachte Berechnung nicht egal. Vereinfacht ist der Querschnitt der Kontakte vernachlässigbar und du rechnest mit , wie ich es auch schon beschrieben habe.
GvC



Anmeldungsdatum: 07.05.2009
Beiträge: 14861

Beitrag GvC Verfasst am: 09. Okt 2011 19:07    Titel: Antworten mit Zitat

No_body hat Folgendes geschrieben:
Die Lage der Kontakte ist auch für eine vereinfachte Berechnung nicht egal. Vereinfacht ist der Querschnitt der Kontakte vernachlässigbar und du rechnest mit , wie ich es auch schon beschrieben habe.


Wenn Du mit Vereinfachung meinst, den Übergangsbereich von maximaler zu minimaler Stromdichte zu vernachlässigen, da er nur sehr kurz ist, dann hast Du recht (ich würde das allerdings nicht vernachlässigen wollen). Wenn Du allerdings nur den Widerstand der metallischen Kontakte vernachlässigsen willst, was sowieso immer gemacht wird, dann ist Deine Vereinfachung falsch.

Der tatsächliche Widerstand wird irgendwo zwischen rho*l/A und 2*rho*l/A liegen (mit A=Querschnittsfläche des Zylinders). Wie gesagt, das lässt sich nur mit einem Simulationsprogramm hinreichend genau bestimmen. Es kommt auf den Zusammenhang an, in dem diese Aufgabe gestellt ist. Dazu könnte Grohte sich vielleicht nochmal äußern.
Grohte
Gast





Beitrag Grohte Verfasst am: 09. Okt 2011 19:59    Titel: Antworten mit Zitat

Das verstehe ich jetzt nicht?

GvC
Der tatsächliche Widerstand wird irgendwo zwischen rho*l/A und 2*rho*l/A liegen
_____________________________

GvC
Zusammenhang an, in dem diese Aufgabe

Widerstand verändern

konstanten,
zylindrischen Widerstand
p =10^5 Ohm
L =0,1 Meter
A =7,854 m^2 (0,01m*0,01m*pi/4)

variable
kontaktfläche aus Aluminium
kontaktfläche von A*0 bis A des zylinder Widerstandes

und ich nur R = p*L/A gefunden habe wuste ich nicht was ich nun nehem musste, wie am anfang erwähnt
GvC



Anmeldungsdatum: 07.05.2009
Beiträge: 14861

Beitrag GvC Verfasst am: 09. Okt 2011 20:01    Titel: Antworten mit Zitat

Grohte hat Folgendes geschrieben:
Zusammenhang an, in dem diese Aufgabe

Widerstand verändern

konstanten,
zylindrischen Widerstand
p =10^5 Ohm
L =0,1 Meter
A =7,854 m^2 (0,01m*0,01m*pi/4)

variable
kontaktfläche aus Aluminium
kontaktfläche von A*0 bis A des zylinder Widerstandes

und ich nur R = p*L/A gefunden habe wuste ich nicht was ich nun nehem musste, wie am anfang erwähnt


Mit diesem Satzbau komme ich nicht klar, geschweige denn mit dem Inhalt.
Grohte
Gast





Beitrag Grohte Verfasst am: 09. Okt 2011 20:42    Titel: Antworten mit Zitat

entschuldigung

der Widerstand soll variabel sein aber mit den gegebenan Dingen von dehnen

zylindrischen Widerstand
p =10^5 Ohm
L =0,1 Meter
A =7,854 m^2 (0,01m*0,01m*pi/4) kontaktfläche

nicht verändert wedern können

so bleibt nur nach der Formel: R = p*L/A , A als variable übrig

die kontaktfläche der Kontakte ( im verhältniss zur konstanten kontaktfläche des zylindrischen Widerstand ) läst sich verändern von A*0 bis A

wuste ich nicht was ich nun nehem musste, wie am anfang erwähnt, mir scheint es nur so alsob nur die veränderrung der Länge des Widerstandes möglich währe
GvC



Anmeldungsdatum: 07.05.2009
Beiträge: 14861

Beitrag GvC Verfasst am: 10. Okt 2011 01:00    Titel: Antworten mit Zitat

Der Satzbau ist zwar immer noch ziemlich unverständlich, aber kommen wir mal zum Inhalt:

Was soll z.B. das hier sein?

Grohte hat Folgendes geschrieben:
p =10^5 Ohm


Dem Größensymbol nach soll es sich wohl um einen spezifischen Widerstand handeln, da das griechische rho von Leuten, die dem Formeleditor misstrauen, häufig durch p ersetzt wird. Aber dann stimmt die Einheit nicht. Der Einheit nach ist es ein Widerstand, aber das ergibt keinen Sinn.

Oder das hier?

Grohte hat Folgendes geschrieben:
A =7,854 m^2 (0,01m*0,01m*pi/4) kontaktfläche


Da passt nun überhaupt nichts zusammen, zumindest wenn man annimmt, dass die Angaben in der Klammer der Erläuterung dienen sollen. Jede Menge Fragen:

1. Woher kommen die 0,01m? Das sieht nach einem Durchmesser aus, die Formel ist die einer Kreisfläche.
2. Wie kommst Du bei einem Kreisdurchmesser von 1cm auf eine Fläche von fast 8m²? Bei mir käme dabei ein Hunderttausendstel der von Dir angegebenen Fläche raus.
3. Wenn das die Kontaktfläche ist und die konstant sein soll, was ist dann die Aufgabenstellung? Wieso sagst Du die Kontaktfläche bliebe als Variable übrig, wenn Du gerade vorher gesagt hast, die angegebenen Größen könnten nicht verändert werden?
4. Dann spekulierst Du über eine veränderbare Länge, die Du auch gerade zuvor als unveränderlich bezeichnet hast. Was meinst Du nun eigentlich?

Ich könnte mir diese Fragen alle selbst beantworten oder brauchte sie gar nicht erst zu stellen, wenn Du eine eindeutige Aufgabenstellung zitieren (Abschreiben kann doch nicht so schwierig sein) und vielleicht sogar eine Skizze liefern könntest. Mit den bisherigen Angaben basierte alles, was ich als Hilfe anbieten könnte, auf reiner Hellseherei.


Zuletzt bearbeitet von GvC am 11. Okt 2011 10:52, insgesamt einmal bearbeitet
Grohte
Gast





Beitrag Grohte Verfasst am: 10. Okt 2011 18:57    Titel: Antworten mit Zitat

der Formeleditor ist nicht so toll es sieht zwar danach gut aus ist mir persöhnlich zu umstendlich, da beim nacheinander eingeben (lings-> rechs) es rückwerz angezeigt wird (rechs-> lings) wo bekomme ich denn die anderen Zeichen her.

es müsste eine Liste geben mit allen Zeichen, die man nur durch anklicken in seine folmel ein setzten kann (rückgenging machbar bis 5-mal einsetzen) und die formel dann auch von (lings-> rechs) nach jedem einsetzen angezeigt wird und wenn man vertig ist wird sie mit nur einem OK-klick in die Antwort eingefügt

1.ist der Durchmesser ist ja auch ein zylindrischen Widerstand

2.ich habe in Milimetern gerechnet und Meter hingeschrieben Hammer

3.in den angegebenen Größen ist nicht Kontaktfläche der Kontakte als konstand angegeben nur die Kontaktfläche des Widerstandes

Die Kontaktfläche der Kontakte läst sich von

Kontaktfläche des Widerstandes*0 = 0 Kontaktfläche der Kontakte
bis
Kontaktfläche des Widerstandes = Kontaktfläche der Kontakte
vareiren

von der Fläche die die Kontaktfläche der Kontakte die Kontaktfläche des Widerstandes abdecken
0 -> Kreisabschnitt-> Kreis

4.ich habe nie über eine veränderbare Länge spekuliert, die Länge des Widerstandes ist als konstant festgelegt und kann nicht verändert werden
GvC



Anmeldungsdatum: 07.05.2009
Beiträge: 14861

Beitrag GvC Verfasst am: 11. Okt 2011 11:01    Titel: Antworten mit Zitat

Beim Lesen Deiner Nachricht macht man sich ja die Augen schmutzig! Das versteht keiner! Hast Du schon mal was von deutscher Rechtschreibung gehört?

Der Inhalt deiner Antworten ist genauso konfus. Deshalb mein Vorschlag zur Güte: Stell' die Aufgabe mal im originalen Wortlaut hier vor, dann sehen wir weiter.
Grohte
Gast





Beitrag Grohte Verfasst am: 11. Okt 2011 18:46    Titel: Antworten mit Zitat

Es ist doch in Deutsch nur nicht halt absolut perfekt Big Laugh

Die Aufgabe, es war nur die Frage am Anfang

und zum besseren verstendnis habe ich Maße mit da zu gegeben um es besser zu illustrieren nur tauchen ab und an Fehler auf (niemand ist perfekt)

den Formeleditor sollte man verbessern und vereinfachen

Anfangsfrage in kurzfassung allgemeingefast

Wiederestandsberechung:

Vorgegeben R= rho*L/A:
- spezifischer elektrischer Widerstand; rho
- Länge; L
- Fläche; A

als veränderlich ist nur die Kontaktfläche der Kontakte die die Elektrische Leitung mit den Wiederstand verbindet ähnlich einem Gleitkontakt, nur mit konstanter Länge.

Bleibt der Wiederstand gleich groß egal wie weit die Kontaktfläche die Fläche des Wiederstandes bedeckt??
GvC



Anmeldungsdatum: 07.05.2009
Beiträge: 14861

Beitrag GvC Verfasst am: 12. Okt 2011 00:56    Titel: Antworten mit Zitat

Also fangen wir mal mit dem ersten Fehler an.Ich habe zwar mehrfach darauf hingewiesen, Du scheinst aber nicht darauf eingehen zu wollen. Das musst Du aber, sonst kann Dir keiner helfen.

Welcher spezifische Widerstand rho ist denn überhaupt gegeben?

Du schreibst

rho = 10^5 Ohm

Das kann nicht sein. Ohm ist die Einheit eines Widerstandes. Der spezifische Widerstand hat die Einheit Ohm*m oder Ohm*cm oder Ohm*mm oder Ohm*mm²/m. Mit welcher Einheit ist der spezifische Widerstand in Deiner Aufgabe gegeben? Das wird sich doch wohl noch abschreiben lassen.

Und dann:

- Ist die Länge tatsächlich 10cm (=0,1m)?
- Ist der Durchmesser des Kreiszylinders tatsächlich 1cm (in der von Dir vorgelegten Aufgabenstellung war von einem Durchmesser nie die Rede)?
Grohte
Gast





Beitrag Grohte Verfasst am: 12. Okt 2011 19:08    Titel: Antworten mit Zitat

Ich bin kein Elektriker und habe auch nicht studiert,
ich arbeite im Einzelhandel

die Länge ist doch richtig 0,1m = 10cm = 100mm
du kannst auch eine andere Länge wählen, meine Maße dienten nur zur besseren illustration

das sieht man doch eigendlich schon an der verwendeten Flächenformel
A =d^2*pi/4 =r^2*pi
das nur der Durchmesser A =d^2*pi/4 gemeit seihen kann

SO?

rho= 10^5 Ohm*mm^2/m
Durchmesser= 0,01m =10mm
Länge =0,1 m

R = p*L/A
10^5 Ohm*mm^2/m*0,1 m*10mm*10mm*pi/4=78539,816 Ohm
GvC



Anmeldungsdatum: 07.05.2009
Beiträge: 14861

Beitrag GvC Verfasst am: 13. Okt 2011 00:18    Titel: Antworten mit Zitat

Jetzt sind wie seit 2 Seiten zugange, und endlich kommst Du erstmals mit vernünftigen Daten für den zylinderförmigen Widerstand.

Warum Du jetzt allerdings den Widerstand ausrechnest (noch dazu falsch, tatsächlich beträgt er 127Ohm), wo doch die Aufgabenstellung eine ganz andere war, bleibt unverständlich. In Deiner Aufgabenstellung hattest Du etwas von variabler Kontaktfläche gesagt. Das ist mir allerdings noch nicht ganz klar. Eine kleine Skizze wäre da sehr hilfreich.

Übrigens: Unterscheide bitte in Zukunft die Querschnittsfläche des Widerstandes und die Kontaktfläche der anliegenden Elektroden. Wo liegen die nun genau, und in welcher Art und Weise soll die Kontaktfläche veränderlich sein? Eines ist jedenfalls sicher: Bei diesem hohen spezifischen Widerstand kannst Du die Aufweitung der Strömungsfeldlinien getrost vernachlässigen, so dass in die Widerstandsformel für A nur die Kontaktfläche einzugeben ist. Wie sich dann der Widerstand errechnet hängt von der bis jetzt - für mich - noch unklaren Position der Elektroden ab.
Grohte
Gast





Beitrag Grohte Verfasst am: 13. Okt 2011 19:35    Titel: Antworten mit Zitat

- wenn ich es mehrfach mache (hir zu fragen) und mich mit den ganzen elektriksachen auskennen würde, dann wäre es nach der ersten Antwort wohl fertig

- eine Zylindrischer Wiederstend, an den Seitenflächen A=d^2*pi/4 sind die Kontakte verschiebar befestigt
von der Fläche die die Kontaktfläche der Kontakte ,AK die Kontaktfläche des Widerstandes,AW abdeckt
0 -> Kreisabschnitt -> Kreis = AK*0 -> Kreisabschnitt, AK -> AW

die Fläche ist doch fast richtig nur noch *10

10^5 Ohm*mm^2/m*0,1 m*10mm*10mm*pi/4=78539,816 Ohm

10^5 Ohm*mm^2/m*0,1 m*10mm*10mm*pi/4=785398,16 Ohm

10^5*0,1 m*10*10*pi/4=785398,16 ohne maßeinheinten

wie kommst du auf 127 Ohm?
GvC



Anmeldungsdatum: 07.05.2009
Beiträge: 14861

Beitrag GvC Verfasst am: 13. Okt 2011 19:51    Titel: Antworten mit Zitat

Grohte hat Folgendes geschrieben:
- eine Zylindrischer Wiederstend, an den Seitenflächen A=d^2*pi/4 sind die Kontakte verschiebar befestigt
von der Fläche die die Kontaktfläche der Kontakte ,AK die Kontaktfläche des Widerstandes,AW abdeckt
0 -> Kreisabschnitt -> Kreis = AK*0 -> Kreisabschnitt, AK -> AW


Das verstehe ich nicht. Bitte Skizze!

Grohte hat Folgendes geschrieben:
10^5*0,1 m*10*10*pi/4=785398,16 ohne maßeinheinten


Ohne Einheiten zu rechnen, ist die größte Todsünde, die man bei physikalischen Berechnungen begehen kann. Sonst hättest Du sofort gemerkt, dass sich bei Dir die Flächeneinheiten nicht rauskürzen, weil Du die Querschnittsfläche im Zähler anstatt im Nenner gerechnet hast.

Grohte hat Folgendes geschrieben:
wie kommst du auf 127 Ohm?


Indem ich alle gegebenen Werte mit ihren Einheiten in die die bekannte Widerstandsformel eingesetzt und das Ganze ausgerechnet habe.
Grohte
Gast





Beitrag Grohte Verfasst am: 13. Okt 2011 20:35    Titel: Antworten mit Zitat

wie stellt man hir eine Skizze hinein, ich würde es über Word zeichnen

wenn mit pi/4 gerechnet wird rechnet man immer mit 0,78539816...., da auch alles 10-er Sachen sind ist es nur mit 10^6*0,78539816 zu nehmen so kommt man nicht auf 127 (mal vor rechnen)

ohne Einheiten kann man überprüfen ob das richtige herauskommt
Grohte
Gast





Beitrag Grohte Verfasst am: 13. Okt 2011 21:23    Titel: Antworten mit Zitat

ich bröchte es nicht zeichen es ist wie:

bei einer totalen Sonnenfinsternis vom beginn (noch keine Sonnenfinsternis) der bis finsterniss hin zur totalen Sonnenfinsternis

Widerstand = Sonne
Kontakt = Mond

eine totale Sonnenfinsternis kann man sich doch gut vorstellen?
GvC



Anmeldungsdatum: 07.05.2009
Beiträge: 14861

Beitrag GvC Verfasst am: 14. Okt 2011 00:59    Titel: Antworten mit Zitat

Grohte hat Folgendes geschrieben:
ohne Einheiten kann man überprüfen ob das richtige herauskommt


Ohne Einheiten kann man das gerade nicht!

GvC hat Folgendes geschrieben:
Sonst hättest Du sofort gemerkt, dass sich bei Dir die Flächeneinheiten nicht rauskürzen, weil Du die Querschnittsfläche im Zähler anstatt im Nenner gerechnet hast.


Warum korrigierst Du Deinen Fehler nicht, wenn Du schon darauf hingewiesen wirst?

Grohte hat Folgendes geschrieben:
wenn mit pi/4 gerechnet wird rechnet man immer mit 0,78539816...., da auch alles 10-er Sachen sind ist es nur mit 10^6*0,78539816 zu nehmen so kommt man nicht auf 127 (mal vor rechnen)


Also gut, ich rechne mal in ganz, ganz kleinen Schritten vor:



Dabei ist





Einsetzen:



Da kürzt sich die Einheit m (Meter) im Zähler raus:



Und da kürzt sich, o Wunder, auch noch die Einheit mm² raus, und übrig bleibt im Zähler die Einheit , die man auch getrost hinter den Bruchstrich schreiben darf.



Das lässt sich nun schon fast im Kopf ausrechnen:
10^5*0,1=10^4
Das muss durch ungefähr 3*25, also ungefähr 75 dividiert werden. Nur um die Größenordung abzuschätzen und sie von Deinem Phantasieergebnis zu unterscheiden, dividieren wir mal durch 100. Dann kommt dabei für den Zahlenwert 100 raus, der noch mit der Einheit multipliziert werden muss. Da wir aber durch weniger als 100 dividieren müssen, ergibt sich ein Ergebnsi von mehr als 100Ohm.

Wenn Du die drei Zahlenwerte in Deinen Rechner eingibst, wird der Dir den Zahlenwert 127,32 (und noch ein paar mehr Dezimalstellen) ausspucken, der, wie gesagt und errechnet, noch mit der Einheit Ohm multipliziert werden muss. Ich habe dann ganz großzügig abgerundet und dabei einen Fehler von 0,25% in Kauf genommen:

Grohte
Gast





Beitrag Grohte Verfasst am: 14. Okt 2011 18:45    Titel: Antworten mit Zitat

da habe ich wohl mit der Fläche statt durch die Fläche gerechnet
Hammer

das richtge ergebnis kann man vielleicht später noch gebrauchen

hast du schon eine Antwort auf:

wenn ich die Kontaktfläche von Kontakt zu Wiederstand ändere, ändert sich dann auch der Wiederstand?? grübelnd

zu besseren Vorstellung:
die Kontaktfläche ändert sich so, als würde der Mond sich vor die Sonne scheiben, solange bis eine totale Sonnenfinsternis entstanden ist.
GvC



Anmeldungsdatum: 07.05.2009
Beiträge: 14861

Beitrag GvC Verfasst am: 15. Okt 2011 15:12    Titel: Antworten mit Zitat

Grohte hat Folgendes geschrieben:
da habe ich wohl mit der Fläche statt durch die Fläche gerechnet


Und da wunderst Du Dich noch? Ich hatte Dir das doch bereits zweimal gesagt.

Auch Deine Frage, ob sich der Widerstand mit der Kontaktfläche ändert, wurde Dir bereits mit Ja beantwortet. Ich habe Dir sogar schon mitgeteilt, dass Du bei diesem relativ hohen spezifischen Widerstand, die Aufweitung der Strömungsfeldlinien vernachlässigen kannst und damit sich der Widerstand errechnet zu



mit
No_body



Anmeldungsdatum: 08.10.2011
Beiträge: 24

Beitrag No_body Verfasst am: 15. Okt 2011 15:54    Titel: Antworten mit Zitat

Wie kommst du darauf das durch einen hohen spezifischen Widerstand keine Aufweitung der Strömung statt findet? Bei der Aufgabe hat man doch einen sehr langen Widerstand von 10 cm. Wenn die Kontaktflächen - von denen ich ausgehe das sie am Anfang und am Enden des Widerstands befestigt sind - ein A_k < A haben, dann werden sich die Elektronen nach einer ziemlich kurzen Strecke (einige µm) gleichmäßig über A aufteilen. Näherungsweise spielt A_k somit keine Rolle, sondern nur A.

Wenn die Kontakte nicht auf den gegenüberliegenden Seiten angebracht sind sondern auf zwei nebeneinander kann es aber wieder ganz anders aussehen. Wurde schon mal genau beschrieben wie die Anordnung aussieht?

Sind nur ein paar Anmerkungen.
GvC



Anmeldungsdatum: 07.05.2009
Beiträge: 14861

Beitrag GvC Verfasst am: 15. Okt 2011 17:19    Titel: Antworten mit Zitat

Das war ja meine anfängliche Argumentation, dass sich die Feldlinien aufweiten. Allerdings nicht so, dass sie nach einigen µm bereits über den ganzen Querschnitt gleichmäßig verteilt sind. Zeichne Dir mal ein quantitatives Feldbild (gleiches Kantenverhältnis der rechteckähnlichen Schnittfiguren aus Äquipotential- und Feldlinien), dann siehst Du, was ich meine. Die Längenangabe war nach Grohtes eigenen Worten ja nur ein Beispiel. Je geringer die Länge, desto weniger ist die Feldlinienaufweitung zu berücksichtigen. Wie das im ganz konkreten Fall mit ganz konkreten Abmessungen aussieht, kann ohnehin nur mit einem Feldberechnungsprogramm ermittelt werden. Geschlossen berechnen lässt sich das jedenfalls nicht.
Grohte
Gast





Beitrag Grohte Verfasst am: 15. Okt 2011 21:13    Titel: Antworten mit Zitat

Als mein Fazit, da das ihr GvC noch langsam wohl in Wissenssachen vordringt von dehnen ich nichs verstehe grübelnd

ist mein Fazit:

egel wie sich die Kontaktfläche verändert (zur besseren illustration stelle man sich eine entstehende totale Sonnenfinsternis vor) der Widerstand bleibt gleich so lange sich nicht die Länge oder spezifischen Widerstand verändert.

Eins hätte ich da noch (da es mich doch langsam interesiert)

No_body:
Wenn die Kontakte nicht auf den gegenüberliegenden Seiten angebracht sind sondern auf zwei nebeneinander kann es aber wieder ganz anders aussehen. Wurde schon mal genau beschrieben wie die Anordnung aussieht?

wenn sie neben einer sind, ( Widerstand sei ein Band, das Band sei sehr dünn) wie würde sich da die Länge berechen, da sie doch auf einer Fläche sind? R = x/A ?
No_body



Anmeldungsdatum: 08.10.2011
Beiträge: 24

Beitrag No_body Verfasst am: 18. Okt 2011 11:21    Titel: Antworten mit Zitat

Wenn man die Kontakflächen verändert und die Länge des zylindrischen Widerstandes ist verhältnismäßig groß, dann bleibt der Widerstand annähernd gleich.

Verändert man die Länge, den spezifischen Widerstand oder die Querschnittsfläch des zylindrischen Widerstands, dann ändert sich der Widerstand natürlich signifikant.

Wenn die Kontakte an zwei nebeneinanderligenden Seiten A und B liegen und die Seite A ist deutlich größer als B, dann verändert man durch die Position des Kontakts an Seite A die wirksame Länge am zylindrischen Widerstand und erhält somit einen anderen Widerstand. Kann man die Position von Kontakt A variieren, dann hat man einen veränderbaren Widerstand (Potentiometer) vorliegen.
Grohte
Gast





Beitrag Grohte Verfasst am: 18. Okt 2011 21:38    Titel: Antworten mit Zitat

Wie berechnet sich solch ein Potentiometer??
No_body



Anmeldungsdatum: 08.10.2011
Beiträge: 24

Beitrag No_body Verfasst am: 20. Okt 2011 10:58    Titel: Antworten mit Zitat

Wenn die Länge sich ändern kann, dann hat man einfach eine veränderliches L, statt einem festen. Die Widerstandsberechnung bleibt natürlich gleich.

Schau mal hier -> http://www.riedl-electronic.at/onlineshop/catalog/images/21160101.JPG

In der oberen Abbildung kannst du oben an dem Widerstand ganz gut einen Kontakt erkennen. Dieser ist über den Sliding Travel verschiebbar. Dadurch änder sich also das L.
Grohte
Gast





Beitrag Grohte Verfasst am: 20. Okt 2011 18:33    Titel: Antworten mit Zitat

Da sehe ich nicht so richtig durch (was sollen die Zaheln, A, B, E, S und Z sein) der beweglich Hebel könnte dann L mit der 6 sein, Sliding Travel ? und lesen kann ich es auch nicht. traurig
No_body



Anmeldungsdatum: 08.10.2011
Beiträge: 24

Beitrag No_body Verfasst am: 20. Okt 2011 19:53    Titel: Antworten mit Zitat

Das sollte eigentlich nur helfen und dich nicht verwirren. Schau dir nur die obere linke Grafik an. Die Zahlen in der unteren Abbildung sind nur irgendwelche Bohrungen, aber das ist hier nicht wichtig.

Genau, der bewegliche Kontakt ist dieser "Hebel" L mit einer Breite von 6 mm. Dieser lässt sich nach rechts verschieben, nämlich genau über diesen Sliding Travel, dadurch ändert sich die wirksame Länge des Widerstandes.
Grohte
Gast





Beitrag Grohte Verfasst am: 20. Okt 2011 21:07    Titel: Antworten mit Zitat

ich kenne mich mit Elektrosachen nicht aus

wenn alle Zahlen Milimeter sind ist das aber "ganz schön klein" und gau
0. mm

Ich verstehe es so:
R=rho*L/A

dann so:

rho*0/A bis rho*L/A

bei L=0 wäre auch R=0 ???
No_body



Anmeldungsdatum: 08.10.2011
Beiträge: 24

Beitrag No_body Verfasst am: 20. Okt 2011 22:22    Titel: Antworten mit Zitat

ja, wenn L = 0 ist, dann ist der Widerstand nautürlich auch Null, weil dann bleiben ja nur die Kontakte übrig. Das hast du also korrekt verstanden.
Neue Frage »
Antworten »
    Foren-Übersicht -> Elektrik